LSAT and Law School Admissions Forum

Get expert LSAT preparation and law school admissions advice from PowerScore Test Preparation.

 Administrator
PowerScore Staff
  • PowerScore Staff
  • Posts: 8950
  • Joined: Feb 02, 2011
|
#22639
Complete Question Explanation

Must Be True. The correct answer choice is (C).

The stimulus presents a series of observations made by Dr. Khan regarding Professor Burns. An overt argument is never stated. Since the distinction between fact and opinion is crucial on the LSAT, especially in Must Be True stimuli where the correct answer choice must satisfy the strict parameters of the Prove Test, it is important to analyze each claim as follows:
  • Earlier observations showed a comet reservoir far out in the solar system, but recent observations cannot confirm this. (Fact)
  • Professor Burns interprets the recent observations as proof that the comet reservoir doesn’t exist. (Opinion)
  • The recent observations occurred under poor conditions. (Fact)
Note the use of double negatives in the second sentence of the stimulus: According to Prof. Burns, “this non-confirmation is enough to show that the earlier observations are incorrect.” It is imperative to immediately simplify this statement by removing both negations and distill its meaning: Burns doesn’t believe there is a comet reservoir far out in our solar system. However, since the recent observations occurred under poor conditions, it would be reasonable to doubt their significance: while they cannot prove (or disprove) the existence of a comet reservoir either way, they certainly cast doubt on Professor Burns’s conclusion. In other words, such a reservoir could exist. This prephrase agrees with answer choice (C).

Answer choice (A): Whether the recent observations, if made under good conditions, would have provided conclusive evidence of a comet reservoir is impossible to know. Beware of hypotheticals and speculative claims when answering Must Be True questions: more often than not, such claims are impossible to prove.

Answer choice (B): Just because Professor Burns reached an invalid conclusion does not mean that the recent observations actually confirm the earlier ones, i.e. that a comet reservoir exists. As stated earlier, it is impossible to know from the information provided whether or not such a reservoir actually exists. This answer choice conflates fact and opinion, and is therefore incorrect.

Answer choice (C): This is the correct answer choice. All we know from the facts presented in the stimulus is that the recent observations do not provide a suitable basis for rejecting the earlier ones. Professor Burns, who believes otherwise, is clearly wrong in making her assessment. In other words, her claims about the implications of the recent observations is incorrect—a comet reservoir could exist (we just don’t know).

Answer choice (D): As with answer choice (A), we have a hypothetical, speculative claim that cannot be proven with absolute certainty. It is entirely possible, however unlikely, that if the recent observations had been made under good conditions, they would have been enough to disprove the earlier ones.

Answer choice (E): This answer choice contains an exaggeration (“worthless”) that cannot be substantiated with the information provided. Eliminate.
 mokkyukkyu
  • Posts: 97
  • Joined: Aug 17, 2016
|
#28896
Hi,

Actually for some reasons I thought E is moderate and C is an extreme answer...
I thought"incorrect" is too strong, as you said it could exist, but maybe not either, right?
So I thought this means E, worthless, because we cannot decide which is true...so it is meaningless, nonsense.
Could anyone clarify this?
 Adam Tyson
PowerScore Staff
  • PowerScore Staff
  • Posts: 5387
  • Joined: Apr 14, 2011
|
#29444
"Worthless" is pretty extreme! That's saying they have absolutely no value at all, and that is way too far based on this stimulus. All we can conclude here is that Burns' reliance on the observations in question is misplaced, since they were done under poor conditions.

Answer C is not saying that the earlier observations are correct and the recent ones are incorrect - that would indeed be too strong an answer here. Pay close attention to what Burns concluded - she said that the recent observations, which did not confirm the earlier ones, are enough (by themselves) to disprove the earlier ones. That is the conclusion that answer C is saying is incorrect - the current observations are insufficient to disprove the earlier ones because they were done under poor conditions.

Again, it is not about which set of observations is correct. It is about whether the new observations are sufficient to draw any conclusions. Burns says they are, but Khan makes clear that they are not.
 jmramon
  • Posts: 47
  • Joined: Jul 21, 2017
|
#41511
Hi,

I have the same question as mokkyukkyu since I had trouble choosing between these contenders. I see what Adam is saying too though.

Could another way to differentiate between C and E be that C more indicates that the recent observations are inconclusive to confirm the comet reservoir due to the poor conditions when the observations took place (“implications are incorrect” (C), as opposed to the conclusion of Burns is incorrect altogether, they just can’t be confirmed with this observation), whereas E says the observations are worthless which means the observations are useless in any capacity even in situations not trying to verify the comet reservoir?

Sorry for the wordyness, but hopefully this further clarifies the distinction in another correct manner. Please let me know if this also works. Thanks!
 Claire Horan
PowerScore Staff
  • PowerScore Staff
  • Posts: 408
  • Joined: Apr 18, 2016
|
#42089
Hi jmramon,

I don't see the first reason you gave as at all different from Adam's, which is great because it is right! It's always helpful to rephrase the reasoning in your own words.

Your second reason is also helpful as a way to be sure about eliminating E: "E says the observations are worthless which means the observations are useless in any capacity even in situations not trying to verify the comet reservoir?" Answer choice E does not clarify "worthless" to mean "worthless for purposes of observing the comet...", so "worthless" would mean without worth at all.

Thanks for your comment!
 lsatretaker
  • Posts: 17
  • Joined: Apr 13, 2019
|
#65038
Hello,

I selected the right answer, but only after going back when I finished the section and changing my answer. I understand what the explanation is saying about strength, which definitely contributed to my decision to change my answer. But I think what helped most was pretending like I was drawing a conclusion from the fact set. Obviously, this wouldn't work on every MBT/MSS question, but is that line of thinking on the right track in general for these questions?

Thanks!
 James Finch
PowerScore Staff
  • PowerScore Staff
  • Posts: 943
  • Joined: Sep 06, 2017
|
#65398
Hi Retaker,

Absolutely! Most MBT/MSS questions don't contain a conclusion in the stimulus, and even those that do are asking for an inference/conclusion to be drawn (often from the stated conclusion). These types are among the most susceptible to a good Prephrase, so make sure that you're taking the time to try and anticipate an answer before diving into the answer choices.

Hope this helps!
 Mozart
  • Posts: 7
  • Joined: Apr 16, 2020
|
#74971
Hello,

When I read through C), I thought that Professor Burn's claim could still have a slim chance of being correct despite the poor conditions, even if the conditions do cast doubt on her conclusion. Perhaps she could still make reasonable observations, even if the observations weren't great.
Would it be fair to say that the stimulus doesn't 100% prove C) to be true, but since the question stem asks for what is "most supported" and not what "must be true," this is okay?


Thank you so much for your help!
User avatar
 KelseyWoods
PowerScore Staff
  • PowerScore Staff
  • Posts: 1079
  • Joined: Jun 26, 2013
|
#74977
Hi Mozart!

You're absolutely right that since this is a Most Strongly Supported question we don't have to hold the answer choice to quite the high standard we would place on it in a Must Be True question. However, it's worth paying close attention to what exactly Professor Burns's claim is. Her claim is that "nonconfirmation is enough to show that the earlier observations are incorrect." So the real issue is NOT whether or not the earlier observations are correct or incorrect--it's whether the recent observations are sufficient to draw a conclusion about the correctness of the earlier observations. If the recent observations occurred under poor conditions, that is enough to show that we can't really draw any definitive conclusions from them and, thus, Professor Burns's claim that we can draw a definitive conclusion from them is incorrect.

Hope this helps!

Best,
Kelsey
User avatar
 attorneyatpaw
  • Posts: 13
  • Joined: Oct 18, 2024
|
#110045
I selected answer choice (D) and don't understand why it's incorrect. Doesn't (D) describe a common LSAT flaw where a lack of evidence doesn't prove something is true or false? Even if the observations were made under good conditions, the fact that "the recent observations" failed to confirm the earlier ones wouldn't have been "enough" to conclude that the earlier observations were incorrect.

Get the most out of your LSAT Prep Plus subscription.

Analyze and track your performance with our Testing and Analytics Package.